Errata in the Reprint of the Examkracker's 9th edition MCAT® Manuals

17
The most current reprint of the 9th edition Examkrackers MCAT ® Manuals is now available and Examkrackers has corrected most errata. This document contains the only remaining errata in the Spring 2015 Reprint of the 9th edition Manuals. The Errata Forum no longer applies to this reprint. This document will be updated as any further errata are posted. Please note the document date above. How to know that you have the most recent (Spring 2015) Reprint of the manuals: If your Manuals reflect the following indicators, then you do have the Spring 2015 reprint and can rely only on this document for any errata. (Note: If you purchased the Complete Study Package with all 6 Manuals included, verify only one Manual as follows. If you purchased your manuals individually, please verify each Manual as follows.) Biology 1: Molecules, 9th edition Reprint page 50, question 36, choice A says "eukaryotic ribosomes are smaller." Biology 2: Systems, 9th edition Reprint No changes were made at this reprint. All known errata are included in this document. Physics 9th edition Reprint page 196, chart heading reads “Answers to the 30 Minute In Class Exams” Chemistry 9th edition Reprint page 141, question 81 includes a chart above the formula Reasoning Skills 9th edition Reprint page 160, question 55 the answer should be listed as D not B Psychology & Sociology 9th edition Rep rint page 159, question 16 Answer Explanation should say “the encouragement of healthy behaviors” not “unhealthy”. Errata in the Reprint of the Examkracker's 9th edition MCAT ® Manuals LAST UPDATED 08/11/16

Transcript of Errata in the Reprint of the Examkracker's 9th edition MCAT® Manuals

Page 1: Errata in the Reprint of the Examkracker's 9th edition MCAT® Manuals

The most current reprint of the 9th edition Examkrackers MCAT® Manuals is now available and Examkrackers has corrected most errata.

This document contains the only remaining errata in the Spring 2015 Reprint of the 9th edition Manuals. The Errata Forum no longer applies to this reprint.

This document will be updated as any further errata are posted. Please note the document date above.

How to know that you have the most recent (Spring 2015) Reprint of the manuals:

If your Manuals reflect the following indicators, then you do have the Spring 2015 reprint and can rely only on this document for any errata.

(Note: If you purchased the Complete Study Package with all 6 Manuals included, verify only one Manual as follows. If you purchased your manuals individually, please verify each Manual as follows.)

Biology 1: Molecules, 9th edition Reprintpage 50, question 36, choice A says "eukaryotic ribosomes are smaller."

Biology 2: Systems, 9th edition Reprint No changes were made at this reprint. All known errata are included in this document.

Physics 9th edition Reprint page 196, chart heading reads “Answers to the 30 Minute In Class Exams”

Chemistry 9th edition Reprint page 141, question 81 includes a chart above the formula

Reasoning Skills 9th edition Reprintpage 160, question 55 the answer should be listed as D not B

Psychology & Sociology 9th edition Rep rintpage 159, question 16 Answer Explanation should say “the encouragement of healthy behaviors” not “unhealthy”.

Errata in the Reprint of the Examkracker's 9th edition

MCAT® ManualsLAST UPDATED 08/11/16

Page 2: Errata in the Reprint of the Examkracker's 9th edition MCAT® Manuals

PHOTOCOPYING & DISTRIBUTION POLICYThe illustrations and all other content in this book are copyrighted material owned by Osote

Publishing. Please do not reproduce any of the content, illustrations, charts, graphs, photos, etc., on email lists or websites.

Photocopying the pages so that the book can then be resold is a violation of copyright.

Schools and co-ops MAY NOT PHOTOCOPY any portion of this book. For more information, please contact Osote Publishing: email: [email protected] or

phone 1.888.KRACKEM.

No part of this work may be reproduced or transmitted in any form or by any means, electronic or mechanical, includ-ing photocopying, and recording, or by any information storage or retrieval system without prior written permission of the copyright owner unless such copying is expressly permitted by federal copyright law, or unless it complies with the Photocopying & Distribution Policy listed above.

© 2015 Osote PublishingAll rights reserved.

Page 3: Errata in the Reprint of the Examkracker's 9th edition MCAT® Manuals

ERRATA LISTED BY BOOKBIOLOGY ILecture 1, page 25:

Figure 1.17, see figure to the side, where the Km of non-competitive inhibition corresponds to ½ V1max.

Lecture 2, page 47:

In the middle of the last paragraph, it should say "DNA polymerase" not "RNA polymerase" as follows: "As is shown in Figure 2.15, at each replication fork, one of

the separated strands is able to be read continuously in the 3´ → 5´ direction while DNA polymerase continuously synthesizes a complementary strand in the 5´ → 3´ direction".

Lecture 2, page 52, figure 2.19:

The middle image on the left should have the following caption: "Addition/deletion - continuing protein

The addition (or deletion) of a nucleotide causes a frameshift. From that point forward, codons are translated on a different reading frame, resulting in different amino acids. If an early stop codon is not introduced, a whole protein is translated. This type of mutation usually results in non-functional proteins."

The image on the bottom left corner of the page should have the following caption: "Deletion/addition - truncated protein (nonsense)

The deletion (or addition) of a nucleotide causes a frameshift. From that point forward, codons are translated on a different reading frame, resulting in different amino acids. If an early stop codon is introduced, the protein is truncated. This type of mutation usually results in non-functional proteins."

Lecture 3, page 87, question 72:

Answer choice B. should say "34" not "32".

In Class Exams, Lecture 3, pages 138 & 155, question 65:

On page 138, question 65 answer choice A. should say "glycogenesis" instead of "glucogenesis".

On page 155, the last sentence of the answer explanation for question 65 should say, " It is insulin that promotes glycogenesis and glycolysis." - "glycogenesis"not "glucogenesis".

In Class Exams, Lecture 1, page 148, question 10:

In the answer explanation for question 10, the second sentence should read "If the turnover number (the numerator) decreases..." instead of "(denominator)".

1

Page 4: Errata in the Reprint of the Examkracker's 9th edition MCAT® Manuals

In Class Exams, Lecture 3, page 153, question 57:

The answer explanation to question 57 should read: "As shown in Figure 2..." not "Figure 1."

Lecture 3, page 169, question 72:

The answer explanation to question 72 should say, "The net product of glycolysis for one glucose molecule is 2 ATP and 2 NADH. The additional steps of aerobic respiration produce 30 ATP, based on a total of 2 ATP from the Krebs cycle, 3 ATP/NADH and 8 total NADH, and 2 ATP per FADH2 and 2 FADH2. The 2 ATP from glycolysis can be added to arrive at 32 total ATP. Finally, the NADH from glycolysis must be accounted for. They produce 3 ATP each in the electron transport chain, but also require 2 ATP to enter to mitochondrial matrix based on the question stem. This results in an additional 2 ATP to arrive at the final answer of 34 ATP."

Lecture 4, page 172, question 94:

In the second line, the answer explanation for question 94 should read "whereas a gene that is down-regulated will appear red."

2

Page 5: Errata in the Reprint of the Examkracker's 9th edition MCAT® Manuals

BIOLOGY IILecture 3, page 85:

In the second paragraph under "Posterier Pituitary" it should say "When water is absorbed from the collecting tubule, the osmolarity of the blood plasma decreases" not increases.

Lecture 3, page 91:

In the second paragraph, the end of the first sentence should say that aldosterone is secreted "from the adrenal cortex", not from the adrenal medulla.

Lecture 4, page 121:

In the middle of the page it should say, "This equation demonstrates the directly proportionate relationship between pressure and resistance." not "indirectly". The signpost should be to "Fluids".

Lecture 4, page 130:

The second paragraph on the page should be titled "B-cell Immunity" in large print, not "T-cell Immunity".

Lecture 6, page 183:

Near the bottom of the first paragraph, it should say, "By contrast to skeletal muscle, which connects bone to bone via ligaments," not 'tendons'.

In Class Exams, Lecture 5, page 227:

Figure 1 of In-Class Exam Passage III. See figure below which shows the macula densa and juxtaglomerular cells. The caption is correct. The glycolysis reaction should not be here.

In Class Exams, Lecture 6, page 233, question 123:

Question 123, answer choice B should say, "CASQ1 null mice are larger than wild-type mice."

3

Page 6: Errata in the Reprint of the Examkracker's 9th edition MCAT® Manuals

In Class Exams, Lecture 5, page 240 & 257, question 112:

The answer explanation should read "A is correct. Again, aldosterone increases the activity of the Na+-K+-ATPase pump, thereby increasing both Na+ reabsorption and K+ excretion in the distal tubule and collecting duct. Increased K+ excretion, however, does not lead to increased blood pressure. Instead, increased Na+ reabsorption leads to increased thirst and increased blood volume. When Na+ is reabsorbed, water follows. In this way, aldosterone indirectly increases blood pressure. Secretion of vasopressin, the hypothalamic hormone that regulates water reabsorption in the distal tubule and collecting duct is stimulated by Ang II, not aldosterone."

In Class Exams, Lecture 1, page 243, question 20

The answer explanation should read "C is the best answer. As described by the fluid mosaic model, all proteins inserted into the membrane must have a hydrophobic portion that can interact with the hydrophobic lipids that make up the cell membrane. Choice A is not the best answer because K+ channels are not synthesized in the nucleus They are made at the rough endoplasmic reticulum like other membrane-bound or secreted proteins. Choice B is not the best answer because the K+ channels do not 'create' the gradient. They play one part, allowing K+ ions to help set up resting potential. Choice D is not a true statement; the K+ channels do not create energy for the cell."

4

Page 7: Errata in the Reprint of the Examkracker's 9th edition MCAT® Manuals

PHYSICS

Lecture 1, page 3:

The last equation in the cluster of 4 under the figure should read: dbase = 4 m, not dramp = 4 m.

Lecture 2, page 50:

In the section, "The Ramp", in the sentence at the end of the 7th line, should say "Thus the force has been reduced by the machine." not "the work".

Lecture 3, page 62:

The fifth sentence under hydraulic lift should read "Since piston 2 has a greater area than piston 1, and force and area are directly proportional when pressure is constant...." -- 'directly' not 'indirectly' proportional.

Lecture 3, page 64:

See new visual below for Figure 3.6. A for area needs to be added to each of the three formulas for force. It should say FB = ρfluidgAΔy. Buoyant force is ρVg. Here that would be ρfluidg AΔy, as AΔy = V.

Lecture 3, page 67:

The text at the bottom of figure 3.9 should read "Object weight = Apparent weight + Weight lost." The second part should read "Fraction of weight lost = ρfluid/ρobject = mfluid/mobject."

Lecture 3, page 76:

In the third paragraph, the equation for area should be A = 2πrd

5

Page 8: Errata in the Reprint of the Examkracker's 9th edition MCAT® Manuals

Lecture 3, page 79, question 69:

Answer choices C and D should both say “because flow speed is lower” as follows: “C. The pressure is higher in the aneurysm because flow speed is lower in the normal blood vessel.” “D. The pressure is lower in the aneurysm because flow speed is lower in the normal blood vessel.”

Lecture 4, page 102, question 86:

The following parenthetical should be included at the end of the question stem before the colon. (The equation for power is P = IV).

In Class Exams, Lecture 4, pages 103 & 214 & 225, question 87:

On p. 103, answer choice A to question 87 should say "5.5 x 106 ions".

On p. 214, the answer to question 87 should be A.

On p. 225, the answer to question 87 should be A and the answer explanation should read:"Choice A is correct. The total charge that can be stored across a capacitor, like the cell, can be calculated by Q = CV. Next, the surface area of the spherical cell = 4πr2 = 4π × (10-3cm)2 = 1.26 x 10-5 cm2. To find the total capacitance of the cell membrane, multiply capacitance/area by area: C = 1.26 x 10-5 × 10-6 = 1.26 × 10-11 F. The total charge can be found by multiplying capacitance by the voltage given in the text: Q = CV = 1.26 x 10-11 × 70 x 10-3 = 8.82 × 10-13 C. Divide the total charge required by the charge per ion (for potassium, a singly charged ion): 8.82 × 10-13 / (1.6 x 10-19) = 5.5 × 106 ions. Choice A is the best answer."

Lecture 4, page 106, question 90:

The following parenthetical should be included at the end of the question stem before the colon (Power = IV = I2R = V2/R).

Lecture 4, page 106, question 91:

The following should be included at the beginning of the question stem: “Voltage sources for alternating current are characterized by their average voltage, Vrms • (Vrms = Vmax/square root of 2)."

Lecture 5, page 155:

See new visual. (Table 5.4, under Divergers do = f should be included in the smaller block. “No image” refers only to do = f for Convergers.)

TABLE 5.4 > Magnification

Image Size

do Convergers Divergers

do > R Smaller

Smallerdo = R Same size

do < R Larger

do = f No image

6

Page 9: Errata in the Reprint of the Examkracker's 9th edition MCAT® Manuals

Lecture 5, page 159:

The sentence starting 2/3 of the way down in the last paragraph on p. 159 should read: "Because the image formed by the objective falls inside the focal length of the eyepiece, the eyepiece forms a virtual, upright image. Since the first image was inverted, this means that the image viewed by the observer appears inverted. For both microscopes and telescopes, even though the object of the second lens is the image of the first one, the object is still "where it belongs" (on the other side of the lens from the observer and is thus designated positive)."

In Class Exams, Lecture 2, page 173, question 29:

Question 29 should read: "Which position has the greatest torque due to the muscle?"

In Class Exams, Lecture 2, page 174:

The y-axis of Figure 1 should be relabeled "Muscle power (kW)".

In Class Exams, Lecture 3, page 180, question 62:

The question stem should read “pulmonary vein” not “vena cavae”.

In Class Exams, Lecture 4, pages 196 & 208, question 81:

The correct answer is choice C. Both instances of “sin” in the answer explanation should be changed to “cos.” The last two sentences should read: “In this case, that would be x cos(30°). The work done then is qEx cos(30°).”

In Class Exams, Lecture 5, page 210, question 100:

See new visual for the equation in the answer explanation for question 100. The f1 at the start of the right side of the equation should be removed, and /4πr2 should be added to the end of the equation.

In Class Exams, Lecture 5, page 211, question 105:

In the answer explanation to q. 105, starting with the sentence on the 5th line, should say: "Therefore, the wavelengths of waves at the proximal end would be shorter than those at the distal end. Because wavelength is inversely proportional to frequency, the frequency of waves at the proximal end should be higher. Choice C can be eliminated and choice A is the best answer."

Lecture 1, page 216, question 14:

The answer explanation to q. 14 should say: "Horizontal speed has no effect on the length of time that a projectile is in the air, so the 30 m/s of initial horizontal speed can be ignored while solving this problem. Because the initial vertical speed is zero, use the equation x = ½ gt2 to solve for time with x = 40 m and g = 10 m/s2. Another quick approach is to reason that the rock will gain 10 m/s of vertical velocity every second. In this case, the rock goes from 0 m/s to 30 m/s of vertical velocity over 3 seconds. This gives an average vertical velocity of 15 m/s over 3 s, meaning it travels about 45 m downward. This is closest to 40m, making answer choice A the best answer. Choices B, C, and D would all give huge overestimations using this quick method."

Lecture 2, page 218, question 29:

The second sentence of the answer explanation should begin "If the tensions in lines A and B were the only forces acting on the pole,…" not "lines A and C". Also, in the fourth line it should say "counterclockwise" not "clockwise".

7

Page 10: Errata in the Reprint of the Examkracker's 9th edition MCAT® Manuals

CHEMISTRYLecture 1, page 14:

The last sentence of the third paragraph should read: "Since there are two electrons in each orbital, the number of electrons in the periods of the periodic table is 2, 8, 18, and 32." It should say "electrons" not "elements".

Lecture 1, page 36:

In the MCAT Think, Reaction 1 should read:C4H9Cl + OCH3

- ? C5H12O + Cl-

Rate Law under reaction 1: Rate = kf [C4H9Cl][OCH3

-]

Reaction 2 should read: C4H9Cl ? C4H9

+ + Cl- (slow step) C4H9+ = intermediate

C4H9+ + OCH3

- ? C5H12O ( fast step)

Rate Law under reaction 2 should read:

Rate = kf [C4H9Cl]α

Lecture 1, page 38:

The last line of "Salty" text at the bottom of the page should say, "If B is tripled and the rate stays the same, 3? = 1; ? = 0." not "? = 1". "If B is tripled and the rate stays the same, 3? = 1, ? = 0."

Lecture 2, page 57, question 29:

Choice A should say "sp3, sp2, sp2, sp3"

Lecture 2, page 59:

The beginning of the last paragraph on the page beneath the two molecules should read "Cyclohexanol and ethanol…" not "Phenol and ethanol…".

Lecture 2, page 63, question 37:

In the reactant, there should be a hydroxyl coming off of the alpha carbon directly to the left of the carbonyl carbon. In the product, the C=O carbonyl on the bottom of the ring and the O on the bottom of the ring need to be swapped in position.

Lecture 3, page 80:

The last sentence of the second paragraph should say: "Across a row in the periodic table, nucleophilicity decreases…" not 'increases'.

CCH CH2 CH3CH(OH)COH CH3 CH3

OO

C

O

O

O

heat

8

Page 11: Errata in the Reprint of the Examkracker's 9th edition MCAT® Manuals

Lecture 3, page 94:

In Figure 3.25, on the reactant side, remove the bottom hydrogen and the bond which attaches it to the

carbonyl carbon.

Lecture 3, page 110:

The last sentence of the first paragraph in the Proteins and Amino Acids subsection should read: “The reaction is a typical nucleophilic substitution reaction, with -OH as the leaving group.” with -OH in place of "alcohol".

Lecture 3, page 113:

The first sentence in the second paragraph should say “The carbon adjacent to the carbonyl carbon is called the …” adding "carbon adjacent to the"

Lecture 4, page 137:

The equation should say delta Srxn = ΔSproducts - ΔSreactants (Replace the second equal sign with a minus sign).

H

HO C

H

H

H

H H

O O

C C C +OHH

H

HH H HH

HO

OCC C

4-hydroxybutanoic acid y-butyrolactone water

FIGURE 3.25 Formation of a Lactone by Dehydration

9

Page 12: Errata in the Reprint of the Examkracker's 9th edition MCAT® Manuals

Lecture 4, page 149, questions 94, 95, & 96:

Questions 94, 95 and 96 should read:

Questions 94-96 refer to the reaction shown below.

2SO2(g) + O2(g) ? 2SO3(g) ∆H° = -198 kJ

Question 94During the formation of sulfur trioxide, an increase in temperature would result in:

○A. a decrease in the equilibrium concentration of oxygen.

○B. an increase in the equilibrium concentrations of products.

○C. no change in equilibrium concentrations.○D. a decrease in the equilibrium concentration of sulfur

trioxide.

Question 95For the above reaction, which of the following will most increase the production of heat?

○A. high pressure and high temperature.○B. high pressure and low temperature.○C. low pressure and high temperature.○D. low pressure and low temperature.

Question 96If the above reaction was at equilibrium inside a closed container, decreasing the volume of the container would cause which of the following?

○A. An increase in the formation of sulfur trioxide○B. A decrease in the formation of sulfur trioxide○C. No effect on the formation of sulfur trioxide○D. The effect cannot be determined with the

information provided.

Lecture 5, page 159:

The answer in the middle of the page should read: “P = 5 atm” not P = 5L.

Lecture 5, page 165, question 102:

The second sentence of the question stem should read "The initial temperature is 0°C and pressure remains constant."

Lecture 6, page 181:

In the fourth line of the first paragraph under Solution Concentration, this sentence should be removed. "Molarity depends only on the amount of solvent, not the total volume of solution."

Lecture 6, page 200:

In the subsection “The Nernst Equation,” in the second equation it should say “ln” not “log”.

10

Page 13: Errata in the Reprint of the Examkracker's 9th edition MCAT® Manuals

Lecture 6, page 201:

In the Nernst equation it should say “log” not “ln”.

Lecture 6, page 204:

The first line of Raoult's Law should not have a plus sign, but instead should have nothing between the mole fraction χa and partial pressure terms Pa, indicating multiplication. Please see updated visual below.

Lecture 7, page 210:

In the figure at the top left of the page containing two amino acids, at the bottom of the structure of Arginine there should be a double bond between the C & NH2

+.

Lecture 7, page 224:

In the fourth paragraph, within the parentheses, the word 'strong' should be changed to 'weak'. It should say "...For a diprotic acid whose conjugate base is a weak acid..."

Lecture 7, page 231:

In the paragraph right above the table, the second to last sentence, "acidic" and "basic" should be switched. The sentence should read:  “The isoelectric point is the average of the first two pKa values for acidic amino acids, and the average of the second and third pKa values for basic amino acids.”

Lecture 7, page 232:

The last sentence, just above the image with many powders, should read: "Thus, x is approximately 5.3 × 10-6, which is also the concentration of OH-. The pOH is therefore 5.3, and the pH = 14 - 5.3 = 8.7."

In Class Exams, Lecture 4, page 261:

The lightest bars in Figure 2 should be labeled as "T∆S," not "-T∆S."

In Class Exams, Lecture 6, page 273, question 125:

Change the word “solution” at the end of the question to “the sample”. Additionally, the 0.001 on the first line should have an M after it to indicate units of concentration.

In Class Exams, Lecture 1, page 284 and 288, question 26:

On page 284, change the answer for question 26 from B to A. On p. 288, the answer explanation for q.26 should read:

"A is correct. Consider the second second step in the illustrated reactions (the salts, not the separated enantiomers). The salts are stereoisomers because they have the same bond-to-bond connectivity, and they must be diastereomers because they can be separated by physical means (crystallization). Notice from the diagram that they are NOT mirror images of each other, and therefore cannot be enantiomers, eliminating choice B. The salts are not structural isomers because they have the same bond-to-bond connectivity, eliminating choice C. There is no plane of symmetry, eliminating choice D."

In Class Exams, Lecture 1, page 285, question 4:

The last sentence of the answer explanation for question 4 should say "Therefore Ne has a smaller atomic radius than Ar."

Raoult's Law

Pv = χa Pa

11

Page 14: Errata in the Reprint of the Examkracker's 9th edition MCAT® Manuals

In Class Exams, Lecture 1, page 286, question 16:

The Answer Explanation for question 16 should reference "Chamber 2" and "Chamber 3", in place of "Chamber 1" and "Chamber 2" respectively.

In Class Exams, Lecture 4, page 299, question 87:

The 5th sentence of the answer explanation to question 87 should read “Figure 1 shows that lower efflux rates are found in knockout mice, so choices B and C can be eliminated. Figure 1 says nothing about lactate influx, and it also shows that lactate flow is actually decreased in a high pH environment.” The 6th sentence “Choice D must be wrong…” needs no change.

Lecture 3, pages 312 & 322, question 62:

On page 312, change the answer for question 62 from A to C. On page 322, question 62 should read "C is correct." Toward the end of the second line it should say “Choices A and B contain ketones… Choice D contains an ester… Only choice C contains two alcohols.”

Lecture 4, pages 312 & 326, questions 94-96: On page 312, answers 95 and 96 should be B & A respectively. On page 326, the answer explanations for

questions 94, 95, and 96 should read:

94. D is the best answer. D is the best answer. An exothermic reaction releases heat, which is considered a product. Increasing the temperature disturbs equilibrium by increasing the amount of heat on the product side, resulting in a shift toward the reactants. As a result, the concentration of reactants (SO2 and O2) will increase while the concentration of the product will decrease (SO3). Choices A, B, and C are not strong answers.

95. B is the best answer. B is the best answer. In order to increase the production of heat, the reaction must shift towards the right and generate products. Increasing the temperature will cause a leftward shift, which is opposite the desired effect, eliminating choices A and C. The effects of pressure change are dependent on the moles of gas of the reactants vs. products. Looking only at gases (which are all the compounds in this reaction), 3 moles of reactants generates 2 moles of products. A decrease in pressure shifts the reaction toward the side with more moles of gas, in this case, the reactants. Therefore, a high pressure and low temperature will increase the production of heat, eliminating choice D and leaving choice B as the best answer.

96. A is the best answer. Decreasing the volume causes an increase in pressure (PV = nRT). An increase in pressure favors a shift towards the side with less moles of gas, which are the products in this reaction. As a result, the formation of sulfur trioxide will increase.

Lecture 2, page 317, question 29:

In the answer explanation for question 29, the sentence beginning with “Atom 3” should read “Atom 3 has 3 σ bonds and a lone pair that is conjugated with the π bond in the carbonyl. This results in a nitrogen with sp2 hybridization. Atom 4…” Additionally, the caption pointing to the nitrogen should change to say “sp2 trigonal planar”.

Lecture 3, page 322, question 102:

The first sentence of the Answer Explanation for question 61 should read “Amines typically act as bases or nucleophiles." Remove the word "weak".

Lecture 5, page 327, question 102:

The equations in the answer explanation should now read:

50 J = P(Vf – Vi) = P[nRTf/P – nRTi/P] = nR(Tf – Ti) = (1 mol)(8.314 J/mol K)(273 K – Tf)

Tf = 273 – 50 J/(8.314 J/K) = 267 K = –6o C

12

Page 15: Errata in the Reprint of the Examkracker's 9th edition MCAT® Manuals

Lecture 6, page 330, question 125:

The Answer Explanation for In Lecture question 125 should have the last sentence as follows: “Finally, adding a non-volatile solute decreases the vapor pressure of the whole solution.”

Lecture 6, page 332, question 139:

The answer explanation for question 139 should read: “An electrolytic cell has a negative potential while a galvanic cell has a positive potential. Thus, choice D is the best answer.”

Lecture 7, page 333, question 150:

The Answer Explanation for In Lecture question 150 should read: "... and a concentration of 1.0 × 10-6 mole/L would result in a pH of 6."

13

Page 16: Errata in the Reprint of the Examkracker's 9th edition MCAT® Manuals

REASONING SKILLSLecture i, page 3:

In the fifth paragraph it should say "20% of the questions", not "50%".

Lecture i, page 8:

In the 5th paragraph it should say, "Thus 2.22" not "Thus 2.22".

Lecture i, page 11:

In the second paragraph, it should say "0.00000000000000000000000000199".

Lecture i, page 18:

In the last line, it should say "What if you made the hose larger by increasing the radius?"

Lecture i, page 25:

In the second paragraph, it should say, "Likewise, 1 meter/100 centimeters is the same thing as 1.", not "1 centimeter".

Lecture i, page 26:

The equation for Joules in the middle of the page has a missing equal sign. See new visual below.

Lecture ii, page 29:

In the first sentence on the page it should say "About twenty percent of questions", not "fifty percent".

Lecture ii, page 37:

In the last sentence of the second to last paragraph, it should say "the incidence" not "prevalence" as follows: "Thus, in this example, the negative control experiment strengthens the argument that any change in the incidence of cancer in the coffee group is due to the coffee and not to other factors."

Lecture ii, page 39:

Figure ii.4.C should have the x-axis labeled "Score" and the y-axis labeled "Frequency".

In Class Exams, page 156, question 98:

In question 98, the question stem should reference paragraph 7, not paragraph 6.

In Class Exams, page 160:

The column headings should be ii, 1, 2, 3, 4.

14

Page 17: Errata in the Reprint of the Examkracker's 9th edition MCAT® Manuals

PSYCHOLOGY & SOCIOLOGYLecture 5, page 109:

The first Salty text in the margin should read "You should also review sections 2.9 and 2.10 in the Biology 2 Systems Nervous System Lecture, which discuss visual processing in the eye and nervous system."

Lecture 5, page 110:

In the middle of the second to last paragraph, the two sentences following the bold, red terms should say, "REM sleep is named for the characteristic eye movements associated with it, but more importantly, it is a separate sleep stage of high brain activity. REM sleep does not occur in the first sleep cycle of the night, but makes up a significant portion of sleep in later cycles."

Lecture 5, page 114, question 112:

The question stem should read: "Which of the following would most likely be found in the blood of a sleep-deprived individual at night?"

In Class Exams, Lecture 5, page 172, question 101:

The following sentence can be cut from the answer explanation as the figure is neither given nor necessary to answer the question. "D is also incorrect because Figure 2 shows a DECREASE in active fear response after fear induction among people with the SS genotype."

15